Đến nội dung

Hình ảnh

ÔN THI ĐẠI HỌC 2012


  • Please log in to reply
Chủ đề này có 95 trả lời

#1
Tham Lang

Tham Lang

    Thượng úy

  • Thành viên
  • 1149 Bài viết
Mình thấy, box bất đẳng thức trung học phổ thông có một số topic về BĐT dành cho các bạn ôn thi đại học, nhưng thực sự thì các topic này có rất ít các bạn tham gia nhiệt tình, có rất nhiều lí do làm các bạn không mấy hứng thú lắm, và chúng trở nên cằn cỗi mà dường như không phát triển được là bao. Vì vậy, một topic với sự khoa học và quản lí nhiệt tình sẽ là động lực để thức đẩy các bạn.
Topic nay ra đời, hy vọng là nơi các bạn có thể thoải mái trao đổi, không nhất thiết phải là những bài mang tính cao siêu trong các kì thi HSG, mọi người có gì thắc mắc đều có thể gửi bài. Mình thích một không khí chân tình, thẳng thắn và có những tranh luận sôi nổi.
Trong đó, mình xin được đưa ra một số yêu cầu nhỏ như sau, mong các bạn nhiệt tình chấp thuận :
  • Các đề bài phải rõ ràng, sáng sủa, gõ latex và viết có dấu.
  • Giải như một bài thi, không được nêu chung chung,nếu có thể các bạn hãy nêu hướng làm.
  • Cấm những vụ cãi vã, mà phải thật sự có tinh thần xây dựng topic một cách lành mạnh.
  • Không cho phép những bài toán nhiều hơn 3 biến, những cách giải sử dụng dồn biến (Kiểu đậm chất HSG) S.O.S, $p, q, r$ ... chỉ dành cho các cuộc thi HSG, Ít sử dụng các kí hiệu $\sum, \prod...$ vào bài làm.
  • Khuyễn khích các bài toán mang đậm chất " thi đh" của mấy năm nay, chẳng hạn như dồn về 1 biến (ĐH 2011), sử dụng công cụ hàm số.
Và bây giờ, xin mời các bạn hãy post bài :)

Bài viết đã được chỉnh sửa nội dung bởi WhjteShadow: 10-09-2012 - 17:45

Off vĩnh viễn ! Không ngày trở lại.......


#2
Tham Lang

Tham Lang

    Thượng úy

  • Thành viên
  • 1149 Bài viết
Để mở đầu, mình xin được giới thiệu với các bạn một số bài toán
Bài 1.[ Thi thử lầ 4 THPT chuyên Lê Hồng Phong- Nam Định]
Cho các số thực $a,b,c$ thoả mãn $0<a,b,c\leq 1$. Chứng minh rằng
$$\left (1+\dfrac{1}{abc}\right )(a+b+c) \ge 3+\dfrac{1}{a}+\dfrac{1}{b}+\dfrac{1}{c}$$
Bài 2.
Chứng minh rằng, với các số thực dương thoả mãn $\left (\dfrac{x+y+z}{2012}\right )^2 \le 4xyz$ ta có :
$$\dfrac{\sqrt{x}}{x+\sqrt{yz}} +\dfrac{\sqrt{y}}{y+\sqrt{zx}}+\dfrac{\sqrt{z}}{z+\sqrt{xy}} \le 2012$$

Bài viết đã được chỉnh sửa nội dung bởi WhjteShadow: 10-09-2012 - 17:18

Off vĩnh viễn ! Không ngày trở lại.......


#3
WhjteShadow

WhjteShadow

    Thượng úy

  • Phó Quản lý Toán Ứng dụ
  • 1323 Bài viết

Để mở đầu, mình xin được giới thiệu với các bạn một số bài toán
Bài 1.[ Thi thử lầ 4 THPT chuyên Lê Hồng Phong- Nam Định]
Cho các số thực $a,b,c$ thoả mãn $0<a,b,c\leq 1$. Chứng minh rằng
$$\left (1+\dfrac{1}{abc}\right )(a+b+c) \ge 3+\dfrac{1}{a}+\dfrac{1}{b}+\dfrac{1}{c}$$

Em xin mở màn t0pic ạ :)
Dự đoán dấu bằng xảy ra tại $a=b=c=1$ và mặt khác ta lại có giả thiết $0<a,b,c\leq 1$.Vì vậy $\frac{1}{a},\frac{1}{b},\frac{1}{c}\geq 1$
Hay ta có:
$$\left(\frac{1}{a}-1\right).\left(\frac{1}{b}-1\right)+\left(\frac{1}{b}-1\right).\left(\frac{1}{c}-1\right)+\left(\frac{1}{c}-1\right).\left(\frac{1}{a}-1\right)\geq 0$$
$$\Leftrightarrow \frac{1}{ab}+\frac{1}{bc}+\frac{1}{ca}+3\geq 2.\left(\frac{1}{a}+\frac{1}{b}+\frac{1}{c}\right)$$
$$\Leftrightarrow a+b+c+\frac{1}{ab}+\frac{1}{bc}+\frac{1}{ca}\geq 2.\left(\frac{1}{a}+\frac{1}{b}+\frac{1}{c}\right)+a+b+c-3\,\,\,(*)$$
Mà mặt khác the0 bất đẳng thức $AM-GM$ ta có: $\frac{1}{a}+a+\frac{1}{b}+b+\frac{1}{c}+c\geq 2+2+2=6$
Vậy nên $2.\left(\frac{1}{a}+\frac{1}{b}+\frac{1}{c}\right)+a+b+c-3\geq \frac{1}{a}+\frac{1}{b}+\frac{1}{c}+3\,\,\,\,\,(**)$
Từ $(*)$ và $(**)$ chúng ta có:
$$a+b+c+\frac{1}{ab}+\frac{1}{bc}+\frac{1}{ca}\geq \frac{1}{a}+\frac{1}{b}+\frac{1}{c}+3$$
$$\Leftrightarrow \left(1+\frac{1}{abc}\right)(a+b+c)\geq 3+\frac{1}{a}+\frac{1}{b}+\frac{1}{c}$$
Vậy ta có Điều phải chứng minh.Dấu bằng xảy ra $\Leftrightarrow a=b=c=1$ $\square$
Chia sẻ 1 chút:Em cũng rất m0ng 1 t0pic được lập ra đúng mục đích của những bạn THPT ham mê bất đẳng thức và những bạn học sinh đang ôn thi ĐH thế này.Những bài toán không quá khó,ý tưởng tr0ng sáng và phù hợp với bất đẳng thức thi ĐH hiện nay.Từ lâu việc làm bất đẳng thức đã được coi là "chơi" bởi vì độ đẹp và khó của nó.Các bạn có thể xem những bài đăng tr0ng ngay b0x BĐT THPT này.Rất nhiều bài phải dùng S.O.S,VornicuSchur,p q r....Những kiến thức đó quá xa với việc thi Đại học và thường làm hoang mang các bạn mới học bất đẳng thức.Phải chăng mọi người đang tự khen,tự sướng với những bài đó mà quên đi mục đích THPT học BĐT là để thi ĐH.......
Chúc t0pic thành công :)
Bài 3.
Ch0 các số thực dương $a,b,c$ và $a+b+c=3$.Chứng minh rằng:
$$\frac{3}{a^2b+b^2c+c^2a}+\frac{4}{abc}\geq 5$$

Bài viết đã được chỉnh sửa nội dung bởi WhjteShadow: 10-09-2012 - 18:03

“There is no way home, home is the way.” - Thich Nhat Hanh

#4
yeutoan11

yeutoan11

    Sĩ quan

  • Thành viên
  • 307 Bài viết

Bài 2.
Chứng minh rằng, với các số thực dương thoả mãn $\left (\dfrac{x+y+z}{2012}\right )^2 \le 4xyz$ ta có :
$$\dfrac{\sqrt{x}}{x+\sqrt{yz}} +\dfrac{\sqrt{y}}{y+\sqrt{zx}}+\dfrac{\sqrt{z}}{z+\sqrt{xy}} \le 2012$$

Bài 2 :
Theo $AM-GM$ ta có :
$x+\sqrt{yz} \geq 2\sqrt[4]{x^2yz}$
Tương tự ta được
$VT \leq \frac{1}{2}(\frac{1}{\sqrt[4]{xy}}+\frac{1}{\sqrt[4]{yz}}+\frac{1}{\sqrt[4]{zx}})$
cần CM $\frac{1}{2}(\frac{1}{\sqrt[4]{xy}}+\frac{1}{\sqrt[4]{yz}}+\frac{1}{\sqrt[4]{zx}}) \leq 2012$
Ta có từ giả thiết
$\frac{3(xy+yz+zx)}{2012^2}\leq \frac{(x+y+z)^2}{2012^2}\leq 4xyz$
$\Leftrightarrow 3(\frac{1}{\sqrt{xy}}+\frac{1}{\sqrt{yz}}+\frac{1}{\sqrt{zx}})\leq3(\frac{1}{x}+\frac{1}{y}+\frac{1}{z})\leq 4.2012^2$
Theo $C.S$
$\frac{1}{4}(\frac{1}{\sqrt[4]{xy}}+\frac{1}{\sqrt[4]{yz}}+\frac{1}{\sqrt[4]{zx}})^2 \leq \frac{3}{4}(\frac{1}{\sqrt{xy}}+\frac{1}{\sqrt{yz}}+\frac{1}{\sqrt{zx}})\leq 2012^2$
vậy $VT \leq 2012$

Bài viết đã được chỉnh sửa nội dung bởi yeutoan11: 10-09-2012 - 18:40

Dựng nước lấy việc học làm đầu. Muốn thịnh trị lấy nhân tài làm gốc.
NGUYỄN HUỆ
Nguyễn Trần Huy
Tự hào là thành viên VMF

#5
triethuynhmath

triethuynhmath

    Thượng úy

  • Thành viên
  • 1090 Bài viết

Bài 3.
Ch0 các số thực dương $a,b,c$ và $a+b+c=3$.Chứng minh rằng:
$$\frac{3}{a^2b+b^2c+c^2a}+\frac{4}{abc}\geq 5$$

Bài này nào:
Cách giải có thể không được tự nhiên lắm:
Ta có:
$3(a^2+b^2+c^2)=(a+b+c)(a^2+b^2+c^2)=a^3+ab^2+b^3+bc^2+c^3+a^2c+(a^2b+b^2c+c^2a)$
Cauchy cho 2 số ta có:
$a^3+ab^2\geq 2ab^2,b^3+bc^2\geq 2b^2c,c^3+ca^2\geq 3c^2a\Rightarrow 3(a^2+b^2+c^2)\geq 3(a^2b+b^2c+c^2a)\Rightarrow \frac{3}{a^2b+b^2c+c^2a}\geq \frac{3}{a^2+b^2+c^2}$
Mặt khác:
$\frac{4}{abc}=\frac{4(a+b+c)}{3abc}=\frac{4}{3}(\sum \frac{1}{ab})\geq \frac{12}{ab+bc+ca}(C.S)$
Vậy ta đưa được bất đẳng thức về chứng minh:
$\frac{3}{a^2+b^2+c^2}+\frac{12}{ab+bc+ca}=3(\frac{1}{a^2+b^2+c^2}+\frac{4}{2(ab+bc+ca)})+\frac{6}{ab+bc+ca}\geq \frac{27}{(a+b+c)^2}+\frac{6}{\frac{(a+b+c)^2}{3}}=3+\frac{6}{3}=3+2=5(Q.E.D)(C.S; ab+bc+ca\leq \frac{(a+b+c)^2}{3})$ Dùng 2 BĐT ấy là ra. ngay $ĐPCM$.

Bài viết đã được chỉnh sửa nội dung bởi triethuynhmath: 10-09-2012 - 18:51

TRIETHUYNHMATH

___________________________

08/12/1997


#6
yeutoan11

yeutoan11

    Sĩ quan

  • Thành viên
  • 307 Bài viết

Bài 3.
Ch0 các số thực dương $a,b,c$ và $a+b+c=3$.Chứng minh rằng:
$$\frac{3}{a^2b+b^2c+c^2a}+\frac{4}{abc}\geq 5$$

Giả sử $c$ nằm giữa $a,b$
Ta có : $a(a-c)(b-c) \leq 0$
$\Leftrightarrow a^2b+c^2a\leq ca^2+abc$$\Leftrightarrow a^2b+b^2c+c^2a+abc\leq c(a+b)^2=4c.\frac{a+b}{2}.\frac{a+b}{2}\leq \frac{4(a+b+c)^3}{27}=4$
$\Leftrightarrow a^2b+b^2c+c^2a\leq 4-abc$
Ta cần CM :
$$\frac{3}{4-abc}+\frac{4}{abc}\geq 5\,\,\,\,\,\,(*)$$
Đặt $abc=t$ với $0<t\leq 1$ thì $(*)\Leftrightarrow \frac{3}{4-t}+\frac{4}{t}\geq 5$
$$\Leftrightarrow 3t+4(4-t)\geq 5t.(4-t)$$
$$\Leftrightarrow 5t^2-21t+16\geq 0$$
$$\Leftrightarrow (t-1)(5t-16)\geq 0$$
Hiển nhiên đúng vì $t \leq 1$
Vậy ta có ĐPCM.
-------------------------------------------------------------------
Chú ý:"Giải như một bài thi, không được nêu chung chung,nếu có thể các bạn hãy nêu hướng làm."

Bài viết đã được chỉnh sửa nội dung bởi WhjteShadow: 10-09-2012 - 19:09

Dựng nước lấy việc học làm đầu. Muốn thịnh trị lấy nhân tài làm gốc.
NGUYỄN HUỆ
Nguyễn Trần Huy
Tự hào là thành viên VMF

#7
T M

T M

    Trung úy

  • Thành viên
  • 926 Bài viết
Bài 4. Cho các số thực dương $a;b;c$ thỏa mãn $ab^2+b^2c+c^2a=3$. Chứng minh rằng

$$\sqrt[3]{a+7}+\sqrt[3]{b+7}+\sqrt[3]{c+7} \leq 2(a^4+b^4+c^4)$$

Đề thi thử trường chuyên Hạ Long - Lần I - 2012


____

Em nghĩ với mục tiêu của topic thì nên ưu tiên những bài trong các đề thi thử đại học hơn là những bài trong sách :)

Bài viết đã được chỉnh sửa nội dung bởi luxubuhl: 10-09-2012 - 19:10

ĐCG !

#8
WhjteShadow

WhjteShadow

    Thượng úy

  • Phó Quản lý Toán Ứng dụ
  • 1323 Bài viết

Bài 4. Cho các số thực dương $a;b;c$ thỏa mãn $ab^2+b^2c+c^2a=3$. Chứng minh rằng

$$\sqrt[3]{a+7}+\sqrt[3]{b+7}+\sqrt[3]{c+7} \leq 2(a^4+b^4+c^4)$$

Đề thi thử trường chuyên Hạ Long - Lần I - 2012

Anh Kiên ơi hình như nhầm giả thiết.Bài này phải là $ab^2+bc^2+ca^2=3$
Nhận thấy đây là 1 bất đẳng thức rất lỏng (Do bậc của VP lớn gấp nhiều lần bậc của VT).Ta sẽ dùng $AM-GM$ điểm rơi để đánh giá:
Để ý điểm rơi của bài toán là $a=b=c=1$,lúc đó thì $\sqrt[3]{a+7}=\sqrt[3]{a+7}=\sqrt[3]{a+7}=2$.
Vì vậy áp dụng bất đẳng thức $AM-GM$ ta có $3.2.2.\sqrt[3]{a+7}\leq a+7+8+8$
Hay là $\sqrt[3]{a+7}\leq \frac{a+23}{12}$ Tương tự và cộng lại ta có $VT\leq \frac{a+b+c+69}{12}$
Nhưng mặt khác lại the0 $AM-GM$ thì $4a\leq a^4+1+1+1$ nên $$VT\leq \frac{a+b+c+69}{12}\leq \frac{a^4+b^4+c^4+285}{48}$$
Vậy công việc còn lại là phải chứng minh:
$$2(a^4+b^4+c^4)\geq \frac{a^4+b^4+c^4+285}{48}$$
$$\Leftrightarrow a^4+b^4+c^4\geq 3$$
Nhưng lại 1 lần nữa the0 $AM-GM$ và điều kiện đề bài ta có $a^4+b^4+b^4+1\geq 4ab^2$.Tương tự cộng lại thì:
$$3(a^4+b^4+c^4)+3\geq 4(ab^2+bc^2+ca^2)$$
$$\Leftrightarrow a^4+b^4+c^4\geq 3$$
Vậy ta có điều phải chứng minh.Dấu bằng xảy ra $\Leftrightarrow a=b=c=1\,\,\,\,\,\,\,\,\,\,\,\,\,\,\,\,\,\square$
Bài 5,
Ch0 các số thực $a,b,c$ không âm thỏa $a+b+c=3$.Chứng minh rằng:
$$a^2+b^2+c^2+abc\geq 4$$

Bài viết đã được chỉnh sửa nội dung bởi WhjteShadow: 10-09-2012 - 20:00

“There is no way home, home is the way.” - Thich Nhat Hanh

#9
Tham Lang

Tham Lang

    Thượng úy

  • Thành viên
  • 1149 Bài viết
Bài 6 [Chế]
Cho $\left\{\begin{array}{1}a,b,c>0 \\ b\ge a\ge c \\ab+bc+ca \le 3 \end{array}\right.$
Chứng minh rằng :
$$P=\dfrac{a}{a^3+a^2b+c^3}+\dfrac{b}{b^3+b^2c+a^3}+\dfrac{1}{ab+2c^2} \ge 1$$
Bài 7.[Chế]
Cho $\left\{\begin{array}{1}a,b,c >0 \\ ab+bc+ca \le 3 \\a\ge b\ge c \end{array}\right.$
Chứng minh rằng :
$$\dfrac{a^4}{b^4+a^3b}+\dfrac{b}{c^4+b^2c^2}+\dfrac{1}{2c^3} \ge \dfrac{3}{2}$$

Off vĩnh viễn ! Không ngày trở lại.......


#10
yeutoan11

yeutoan11

    Sĩ quan

  • Thành viên
  • 307 Bài viết

Bài 5,
Ch0 các số thực $a,b,c$ không âm thỏa $a+b+c=3$.Chứng minh rằng:
$$a^2+b^2+c^2+abc\geq 4$$

Cách 1 :
Trước tiên ta xét bổ đề $a^2+b^2+c^2+2abc+1 \geq 2(ab+bc+ca)$
Theo nguyên lí Dirichle tồn tại $2$ trong $3$ số $a,b,c$ cùng $\geq 1$ hoặc $\leq 1$
Giả sử $(a-1)(b-1) \geq 0$
sử dụng hằng đẳng thức :
$a^2+b^2+c^2+2abc+1 - 2(ab+bc+ca) = (a-b)^2 +(c-1)^2+ 2c(a-1)(b-1) \geq 0$
CM xong bổ đề
Quay lại bài toán
$2(a^2+b^2+c^2)+2abc\geq 8$
Ta có :$2(a^2+b^2+c^2)+2abc=[a^2+b^2+c^2+(a^2+b^2+c^2+2abc+1)]-1\geq [a^2+b^2+c^2+2(ab+bc+ca)]-1=(a+b+c)^2-1=8$
Vậy Ta có $ĐPCM$
Cách 2:Dồn biến
Đặt $f(a,b,c)=a^2+b^2+c^2+abc$
Giả sử $a\geq b \geq c$
$\Rightarrow c\leq 1$
Ta CM: $f(a,b,c) \geq f(\frac{a+b}{2},\frac{a+b}{2},c)$
$\Leftrightarrow a^2+b^2+abc\geq \frac{(a+b)^2}{2}+\frac{(a+b)^2.c}{4}$
$\Leftrightarrow \frac{(a-b)^2}{2}(1-\frac{c}{2})\geq 0$(đúng vì $c\leq 1$)
Ta CM :
$f(\frac{a+b}{2},\frac{a+b}{2},c) \geq 4$
$\Leftrightarrow \frac{(3-c)^2}{2}+c^2+\frac{(3-c)^2c}{4}\geq 4$
$\Leftrightarrow c^3-3c+2\geq 0$
$\Leftrightarrow (c-1)(c^2+c-2)\geq 0$
Đúng vì $c\leq 1$
Vậy ta có ĐPCM

Bài viết đã được chỉnh sửa nội dung bởi yeutoan11: 10-09-2012 - 20:29

Dựng nước lấy việc học làm đầu. Muốn thịnh trị lấy nhân tài làm gốc.
NGUYỄN HUỆ
Nguyễn Trần Huy
Tự hào là thành viên VMF

#11
BlackSelena

BlackSelena

    $\mathbb{Sayonara}$

  • Hiệp sỹ
  • 1549 Bài viết

Trước tiên ta xét bổ đề $a^2+b^2+c^2+2abc+1 \geq 2(ab+bc+ca)$

Trình độ có hạn nên không dám đú đởn ...
Em c/m cái bổ đề đó theo cách khác, có vẻ nhẹ nhàng hơn.
$$a^2+b^2+c^2 + 2abc + 1 \geq 2(ab+bc+ca)$$ (*)
Theo $AM-GM$, ta có:
$$2abc+1 = abc + abc + 1 \geq 3\sqrt[3]{a^2b^2c^2} = \frac{9abc}{3\sqrt[3]{abc}} \geq \frac{9abc}{a+b+c}$$
Dó đó ta cần chứng minh bất đẳng thức:
$$a^2+b^2+c^2 + \frac{9abc}{a+b+c} \geq 2(ab+bc+ca)$$
$$\Leftrightarrow a^3+b^3+c^3 + 3abc \geq ab(a+b) + bc(b+c) + ac(c+a)$$
Luôn đúng theo $Schur$ bậc 3, vậy bđt phụ được c/m
--------------------------------------------------------------------------------------
Quang à thi ĐH không được áp dụng $Schur$ trực tiếp mà phải chứng minh lại nên the0 tổng quan thì $Dirichlet$ nhanh hơn em à :)

Bài viết đã được chỉnh sửa nội dung bởi WhjteShadow: 10-09-2012 - 20:39


#12
Tham Lang

Tham Lang

    Thượng úy

  • Thành viên
  • 1149 Bài viết
Hì, anh xin nhắc với mấy em nhé :D Topic này để "ÔN THI ĐẠI HỌC" nên cố gắng hạn chế đến mức tối đa các BĐT khủng vô nhé. Nếu đi thi ĐH phải cm cái đó thì có vẻ không hợp lí lắm. Nên mọi người cứ thoả sức mà làm theo cách cổ điển nhất nếu có thể nhé. Cảm ơn mọi người đã theo dõi.
Chú ý : Khuyến khích các đề thi thử nhé.

Bài viết đã được chỉnh sửa nội dung bởi Tham Lang: 10-09-2012 - 20:44

Off vĩnh viễn ! Không ngày trở lại.......


#13
Tham Lang

Tham Lang

    Thượng úy

  • Thành viên
  • 1149 Bài viết

Bài 1.[ Thi thử lầ 4 THPT chuyên Lê Hồng Phong- Nam Định]
Cho các số thực $a,b,c$ thoả mãn $0<a,b,c\leq 1$. Chứng minh rằng
$$\left (1+\dfrac{1}{abc}\right )(a+b+c) \ge 3+\dfrac{1}{a}+\dfrac{1}{b}+\dfrac{1}{c}$$

BĐT tương đương
$$\dfrac{(a-1)(ab-1)}{ab}+\dfrac{(b-1)(bc-1)}{bc}+\dfrac{(c-1)(ca-1)}{ca} \ge 0$$
Đúng vì $0<a, b, c\le 1$
Đẳng thức xảy ra khi $a=b=c=1$.
Bài 8.[ Thi thử lần 2 THPT Trung Giã]
Cho các số thực dương $a,b,c$ thoả mãn $abc=1$. Tìm GTLN của :
$$P=\dfrac{1}{\sqrt{a}+\sqrt{b}+2\sqrt{c}+2}+\dfrac{1}{\sqrt{b}+\sqrt{c}+2\sqrt{a}+2}+\dfrac{1}{\sqrt{c}+\sqrt{a}+2\sqrt{b}+2}$$

Bài viết đã được chỉnh sửa nội dung bởi Tham Lang: 10-09-2012 - 21:37

Off vĩnh viễn ! Không ngày trở lại.......


#14
WhjteShadow

WhjteShadow

    Thượng úy

  • Phó Quản lý Toán Ứng dụ
  • 1323 Bài viết
Mình xin gợi ý các bạn chứng minh 2 bài toán 6 và 7 của anh Mít tựa tựa như bài toán sau:(Như là lần đầu anh Mít gợi ý ch0 em vậy :P)
Bài toán gần giống 6,7:
Cho $x,y,z>0$.Chứng minh bất đẳng thức:
$$\frac{1}{x^2+xy}+\frac{1}{y^2+yz}+\frac{1}{z^2+zx}\geq \frac{27}{2(x+y+z)^2}$$

Bài viết đã được chỉnh sửa nội dung bởi WhjteShadow: 11-09-2012 - 19:54

“There is no way home, home is the way.” - Thich Nhat Hanh

#15
WhjteShadow

WhjteShadow

    Thượng úy

  • Phó Quản lý Toán Ứng dụ
  • 1323 Bài viết

Mình xin gợi ý các bạn chứng minh 2 bài toán 6 và 7 của anh Mít tựa tựa như bài toán sau:(Như là lần đầu anh Mít gợi ý ch0 em vậy :P)
Bài toán gần giống 6,7:
Cho $x,y,z>0$.Chứng minh bất đẳng thức:
$$\frac{1}{x^2+xy}+\frac{1}{y^2+yz}+\frac{1}{z^2+zx}\geq \frac{27}{2(x+y+z)^2}$$

Tớ đoán hầu hết các bạn khi nhìn thấy bất đẳng thức này đều nghĩ đến bổ đề
$$\frac{1}{a}+\frac{1}{b}+\frac{1}{c}\geq \frac{9}{a+b+c}$$
Nhưng khi áp dụng vào thì:
$$\frac{1}{x^2+xy}+\frac{1}{y^2+yz}+\frac{1}{z^2+zx}\geq \frac{9}{x^2+y^2+z^2+xy+yz+zx}$$
Và ta cần chứng minh $x^2+y^2+z^2+xy+yz+zx\leq \frac{2}{3}(x+y+z)^2$ hay $x^2+y^2+z^2\leq xy+yz+zx$ nhưng đây lại là 1 bất đẳng thức sai mất rồi.
Lật lại vấn đề,ta nghĩ đến cách chứng minh của $\frac{1}{a}+\frac{1}{b}+\frac{1}{c}\geq \frac{9}{a+b+c}$
Là $\frac{1}{a}+\frac{1}{b}+\frac{1}{c}\geq \frac{3}{\sqrt[3]{abc}}\geq \frac{9}{a+b+c}$
Nghĩa là sử dụng đến 2 lần $AM-GM$ 3 số.Ta sẽ bắt đầu lại bài toán với chỉ 1 lần $AM-GM$.Tức là:
$$\frac{1}{x^2+xy}+\frac{1}{y^2+yz}+\frac{1}{z^2+zx}\geq \frac{3}{\sqrt[3]{xyz(x+y)(y+z)(x+z)}}$$
Nhưng bằng 1 chút tinh tế,ta nhận thấy $xyz(x+y)(y+z)(x+z)=(xy+yz)(xy+zx)(yz+zx)\leq \frac{8(xy+yz+zx)^3}{27}$
Vậy nên $$\frac{1}{x^2+xy}+\frac{1}{y^2+yz}+\frac{1}{z^2+zx}\geq \frac{3}{\sqrt[3]{xyz(x+y)(y+z)(x+z)}}$$
$$\geq \frac{9}{xy+yz+zx}\geq \frac{27}{2(x+y+z)^2}$$
Và ta có ĐPCM :)
XIn lỗi các bạn bài 6 anh Mít gõ sai đề =))
Tham khảo tại:
http://diendantoanho...ac1ac-2c2-ge-1/

Bài viết đã được chỉnh sửa nội dung bởi WhjteShadow: 11-09-2012 - 20:12

“There is no way home, home is the way.” - Thich Nhat Hanh

#16
Ispectorgadget

Ispectorgadget

    Nothing

  • Quản lý Toán Phổ thông
  • 2946 Bài viết
Bài 10: Chứng minh $$\frac{1}{2}(1+\frac{1}{2012})(2+\frac{1}{2012})...(2012+\frac{1}{2012})<(2012)!$$
Đề thi thử chuyên QH 2012

►|| The aim of life is self-development. To realize one's nature perfectly - that is what each of us is here for. ™ ♫


#17
WhjteShadow

WhjteShadow

    Thượng úy

  • Phó Quản lý Toán Ứng dụ
  • 1323 Bài viết

Bài 9 ,tìm MIN $\sqrt{\frac{x^3}{x^3+8y^3}}+\sqrt{\frac{4y^3}{y^3+(x+y)^3}}$
(tài liệu luyện thi ĐH của anh -Dang Thanh Nam thủ khoa đó)

Ta có các nhận xét sau:
$\sqrt{1+8a^3}=\sqrt{(1+2a)(1-2a+4a^2)}\leq \frac{4a^2+2}{2}=2a^2+1$
Và $\sqrt{1+b^3}=\sqrt{(1+b)(1-b+b^2)}\leq \frac{b^2+2}{2}=\frac{b^2}{2}+1$
ÁP dụng vào bài toán với $a=\frac{y}{x}$ và $b=\frac{x+y}{y}$ Ta có:
$$\sqrt{\frac{x^3}{x^3+8y^3}}+\sqrt{\frac{4y^3}{y^3+(x+y)^3}}\geq \frac{1}{2\frac{y^2}{x^2}+1}+\frac{4}{\frac{(x+y)^2}{y^2}+2}$$
$$=\frac{x^2}{x^2+2y^2}+\frac{4y^2}{(x+y)^2+2y^2}\geq \frac{x^2}{x^2+2y^2}+\frac{4y^2}{2x^2+4y^2}=1$$
Do $(x+y)^2\leq x^2+y^2$.Vây ta có ĐPCM.
Dấu bằng xảy ra $\Leftrightarrow x=y$ $\blacksquare$
Bài 11,[Đề thi thử chuyên Hà Tĩnh]
Ch0 $x,y,z\in [0;1]$ .Tìm max của biểu thức:
$$P=(1+xyz).\left(\frac{1}{1+x^3}+\frac{1}{1+y^3}+\frac{1}{1+z^3}\right)$$
Bài 12,[Đề thi thử trường chuyên Ams-Hà Nội]:
Ch0 các số thực dương $x,y,z$ thỏa mãn $x(x-1)+y(y-1)+z(z-1)\leq \frac{4}{3}$.Tìm giá trị nhỏ nhất của:
$$A=\frac{1}{1+x}+\frac{1}{1+y}+\frac{1}{1+z}$$

Bài viết đã được chỉnh sửa nội dung bởi WhjteShadow: 11-09-2012 - 20:53

“There is no way home, home is the way.” - Thich Nhat Hanh

#18
yeutoan11

yeutoan11

    Sĩ quan

  • Thành viên
  • 307 Bài viết

Bài 11,[Đề thi thử chuyên Hà Tĩnh]
Ch0 $a,b,c\in [0;1]$ .Tìm max của biểu thức:
$$P=(1+xyz).\left(\frac{1}{1+x^3}+\frac{1}{1+y^3}+\frac{1}{1+z^3}\right)$$

Ta CM BĐT :
$\frac{1}{1+x^2}+\frac{1}{1+y^2}\leq \frac{2}{1+xy}$ (1)
$\Leftrightarrow (x-y)^2(xy-1)\leq 0$ ( Đúng vì $x,y\in[0;1]$ )
Áp dụng (1) Ta CM với $x,y,z,t\in[0;1]$
$\frac{1}{1+x^4}+\frac{1}{1+y^4}+\frac{1}{1+z^4}+\frac{1}{1+t^4}\leq 2(\frac{1}{1+x^2y^2}+\frac{1}{1+z^2t^2})\leq \frac{4}{1+xyzt}$(2)
Áp dụng (2) cho 4 số: $x,y,z,xyz\in[0;1]$
Ta có :
$\frac{1}{1+x^3}+\frac{1}{1+y^3}+\frac{1}{1+z^3}+\frac{1}{1+xyz}\leq \frac{4}{1+\sqrt[4]{x^3y^3z^3xyz}}=\frac{4}{1+xyz}$
$\Leftrightarrow \frac{1}{1+x^3}+\frac{1}{1+y^3}+\frac{1}{1+z^3}\leq \frac{3}{1+xyz}$
Hay $P\leq 3$
Đẳng thức khi "x=y=z"

Bài viết đã được chỉnh sửa nội dung bởi yeutoan11: 11-09-2012 - 21:06

Dựng nước lấy việc học làm đầu. Muốn thịnh trị lấy nhân tài làm gốc.
NGUYỄN HUỆ
Nguyễn Trần Huy
Tự hào là thành viên VMF

#19
Ispectorgadget

Ispectorgadget

    Nothing

  • Quản lý Toán Phổ thông
  • 2946 Bài viết

Bài 12,[Đề thi thử trường chuyên Ams-Hà Nội]:
Ch0 các số thực dương $x,y,z$ thỏa mãn $x(x-1)+y(y-1)+z(z-1)\leq \frac{4}{3}$.Tìm giá trị nhỏ nhất của:
$$A=\frac{1}{1+x}+\frac{1}{1+y}+\frac{1}{1+z}$$

Trước hết ta cm BĐT sau với $x,y,z>0$ thỏa $x(x-1)+y(y-1)+z(z-1)\leq \frac{4}{3}$ ta có $a+b+c\leq 4$
Chứng minh: $$\frac{4}{3}\geq a(a-1)+b(b-1)+c(c-1)=a^2+b^2+c^2-(a+b+c)=\frac{1}{3}(1+1+1)(a^2+b^2+c^2)-a-b-c\geq \frac{1}{3}(a+b+c)^2-(a+b+c)$$
$$\Rightarrow (a+b+c)^2-3(a+b+c)-4\leq 0 \Leftrightarrow [(a+b+c)+1][a+b+c-4]\leq 0\Leftrightarrow a+b+c\leq 4$$
Áp dụng bất đẳng thức trên
$$A\geq \frac{9}{x+y+z+3}\geq \frac{9}{4+3}=\frac{9}{7}$$


►|| The aim of life is self-development. To realize one's nature perfectly - that is what each of us is here for. ™ ♫


#20
WhjteShadow

WhjteShadow

    Thượng úy

  • Phó Quản lý Toán Ứng dụ
  • 1323 Bài viết
Bài 13.[Đề thi thử THPT Uông Bí]
CH0 2 số dương $x,y$ thỏa mãn $12x^2+2y^2=5$.Chứng minh rằng:
$$x+y+\frac{1}{xy}\geq \frac{7}{2}$$
Bài 14.[ Đề thi thử THPT Chuyên Phan Bội Châu Nghê An]
Ch0 các số thực $a,b,c$ thỏa $0\leq a\leq b\leq c$ và $a^2+b^2+c^2=3$.Tìm GTLN của:
$$A=5a-4abc$$
Bài 15.[Dự bị khối A-2 -2009](Đã có trên diễn đàn nhưng chưa ai trả lời :) )
Cho $x,y,z\in [1;3]$ và $x+y+z=6$. Tìm GTLN của $P=x^3+2y^3+z^3$

Bài viết đã được chỉnh sửa nội dung bởi WhjteShadow: 12-09-2012 - 20:23

“There is no way home, home is the way.” - Thich Nhat Hanh




2 người đang xem chủ đề

0 thành viên, 2 khách, 0 thành viên ẩn danh